CAT Exam  >  CAT Tests  >  Mock Test Series for NMAT  >  NMAT Logical Reasoning MCQ Quiz - 3 - CAT MCQ

NMAT Logical Reasoning MCQ Quiz - 3 - CAT MCQ


Test Description

30 Questions MCQ Test Mock Test Series for NMAT - NMAT Logical Reasoning MCQ Quiz - 3

NMAT Logical Reasoning MCQ Quiz - 3 for CAT 2024 is part of Mock Test Series for NMAT preparation. The NMAT Logical Reasoning MCQ Quiz - 3 questions and answers have been prepared according to the CAT exam syllabus.The NMAT Logical Reasoning MCQ Quiz - 3 MCQs are made for CAT 2024 Exam. Find important definitions, questions, notes, meanings, examples, exercises, MCQs and online tests for NMAT Logical Reasoning MCQ Quiz - 3 below.
Solutions of NMAT Logical Reasoning MCQ Quiz - 3 questions in English are available as part of our Mock Test Series for NMAT for CAT & NMAT Logical Reasoning MCQ Quiz - 3 solutions in Hindi for Mock Test Series for NMAT course. Download more important topics, notes, lectures and mock test series for CAT Exam by signing up for free. Attempt NMAT Logical Reasoning MCQ Quiz - 3 | 40 questions in 38 minutes | Mock test for CAT preparation | Free important questions MCQ to study Mock Test Series for NMAT for CAT Exam | Download free PDF with solutions
NMAT Logical Reasoning MCQ Quiz - 3 - Question 1

In a singing class of 55 students, Amit is ranked 20th from the top. When two boys joined the class, his rank dropped by one. What is his new rank from the bottom?

Detailed Solution for NMAT Logical Reasoning MCQ Quiz - 3 - Question 1

Amit is originally ranked 20th from the top in a class of 55.
When two boys join the class, he is ranked 21st in a class of 57.
Thus, there are 36 students ranked below him. Hence, he is now ranked 37th from the bottom. Hence, option 2.

NMAT Logical Reasoning MCQ Quiz - 3 - Question 2

In each of these questions, there is a statement followed by three courses of action numbered I, II and III. Assuming everything in the statement to be true, which of the three courses of action is it logical to follow?


Statement:

A recent study found that private schools do not necessarily offer better education than public schools, inspite of charging high fees.

 


Course of Action:

I. Private schools should lower their fees.

II. Public schools should hire more teachers.

III. Private schools should strive to increase their standard of education.

Detailed Solution for NMAT Logical Reasoning MCQ Quiz - 3 - Question 2

The issue is not whether the fees of private schools are too high, but whether they offer education of a quality worthy of those fees. So statement I is not the right course of action. Eliminate options 1 and 4.
It is not clear how hiring more teachers would help public schools become better; and in any case, the focus of the issue is on private schools, not public ones. So statement II is also ruled out. Eliminate option 2.
Only statement III is the right course of action, based on the points made above. Eliminate option 5.
Hence, the correct answer is option 3

1 Crore+ students have signed up on EduRev. Have you? Download the App
NMAT Logical Reasoning MCQ Quiz - 3 - Question 3

Answer the following question based on the information given below.

It is essential to take a shower right before using a swimming pool which has been disinfected by chlorine.

Which one of the following statements could have led to the above conclusion?

Detailed Solution for NMAT Logical Reasoning MCQ Quiz - 3 - Question 3

The statement concludes that it is important to shower right before a swim. The probable reasoning for this could be that our bodies have residues which mix with the chlorine in the pool water to create a toxic by-product. Only option 1 pertains to this line of reasoning.
Options 2 and 3 are not in line with the rationale of showering before a swim and hence, can be eliminated.
Endorphins are released within the body and cannot change the nature of the water externally. Hence, option 4 can be eliminated.
Option 5 is beyond the scope of the passage and hence, can be eliminated. Hence, the correct answer is option 1.

NMAT Logical Reasoning MCQ Quiz - 3 - Question 4

Group Question Answer the following question based on the information given below.

Five people - A, B, C, D and E - took part in a car race. The person with a red car was from Spain and came first in the race. B was from Argentina. The person who came fourth was from England. The person who came third had a blue car. The person from India did not come second. C had a black car. D finished ahead of the person with a green car. The person from Italy had a white car and finished behind the person with a blue car. A finished behind E.

 

 

Q. Who among the following was from India?

Detailed Solution for NMAT Logical Reasoning MCQ Quiz - 3 - Question 4

Represent the data for each person in the order: Colour — Country — Position — Name.
The person with a red car was from Spain and came first in the race.

Red -- Spain -- 1st -- ?
B was from Argentina.

? — Argentina — ? B

The person who came fourth was from England.

? — England — 4th — ?
The person from Italy had a white car and finished behind the person with a blue car while the person who came third had a blue car.
So, the person from Italy finished 5th (v The person from England came fourth) White - Italy - 5th - ?
The person from India did not come second.

So, the person from India finished third and had the blue car, while the person from Argentina finished second.
Blue — India — 3rd — ?
Since B is from Argentina and C had a black car, C is from England. So, the person from Argentina had the green car.
D finished ahead of the person with a green car D finished first.
A finished behind E. A finished fifth and B finished third.
Thus, the final combinations are:

Red — Spain — 1st - D.

Green — Argentina — 2nd — B.

Black — England — 4th — C.

White — Italy — 5th — A.

Blue — India — 3rd — E.

Thus, E was from India. Hence, option 3.

NMAT Logical Reasoning MCQ Quiz - 3 - Question 5

Five people - A, B, C, D and E - took part in a car race. The person with a red car was from Spain and came first in the race. B was from Argentina. The person who came fourth was from England. The person who came third had a blue car. The person from India did not come second. C had a black car. D finished ahead of the person with a green car. The person from Italy had a white car and finished behind the person with a blue car. A finished behind E.

 

 

Q. At which position did D finish?

Detailed Solution for NMAT Logical Reasoning MCQ Quiz - 3 - Question 5

Consider the solution to the previous question.
D finished first. Hence, option 4.

NMAT Logical Reasoning MCQ Quiz - 3 - Question 6

Rajbir starts from his home and walks for 15 minutes. He then turns around, immediately takes a left turn and walks for another 10 minutes. If his is now walking towards the south-east, in which direction was he originally walking?

Detailed Solution for NMAT Logical Reasoning MCQ Quiz - 3 - Question 6

Assume that Rajbir was initially going towards the north.
When he turned around, he would be facing the south. Now, when he took a left, he would now have gone towards the east.
So, Rajbir’s original direction (north) is 90° anti-clockwise w.r.t. his new final direction (east).
Since his final direction is actually the south-east, the direction that is 90° anti-clockwise w.r.t. it is the north-east.
Thus, he was originally walking in the north-east direction.
Hence, option 4.

NMAT Logical Reasoning MCQ Quiz - 3 - Question 7

A statement is followed by two assumptions. Consider the statement and the following assumptions and decide which of the assumptions is/are implicit in the statement.

“We are woefully unprepared for any large-scale geographic outage that might take place,” explained Joel Gordes, research director for the U.S. National Power Grid. He said that while some generators and transmission lines probably would survive such a loss, they might not be able to muster enough juice to reboot the grid, which experts call a “black start.”

 

Assumptions:

A. A power failure is expected to occur all over the U.S.

B. The power failure could affect the economy of the country.

Detailed Solution for NMAT Logical Reasoning MCQ Quiz - 3 - Question 7

According to the director of the U.S. National Power Grid, the country is unprepared for a large-scale power outage. This does not imply that a power outage is expected. The director is merely putting forth his analysis of the situation. Hence, assumption A is not implicit. Eliminate options 1, 3 and 5. While no link has been suggested between a power failure and the economy of the country, we can assume that a nation-wide power failure would lead to economic losses and hence, affect the economy of the country. Hence, assumption B is implicit. Eliminate option 4.
Hence, the correct answer is option 2.

NMAT Logical Reasoning MCQ Quiz - 3 - Question 8

In the following question, a word is represented by a particular set of numbers present in the given options. A letter is represented by a two digit number. The tens place digit of this number corresponds to the row and the units digit corresponds to the column in which the letter is present.

For example, S can be represented by the number 32 as it is present in the third row and the second column in matrix I. Similarly S can take following values also: 13, 43, 66, 69, 78 and 89.


Find the correct code for the word ABDUCT. 

Detailed Solution for NMAT Logical Reasoning MCQ Quiz - 3 - Question 8

If we look at the data carefully, no letter will have code 93.
Thus, option 1 can be eliminated safely.
From Matrix I, A can be coded as 14, 21 and 33.
From Matrix II, A can be coded as 55, 67, 85 and 88.
Codes for B: 02, 20, 34, 41, 65, 79 and 97 As option 2 has 43 in the 2nd place, it can be eliminated.
Codes for D: 04, 11, 22, 40, 44, 59, 75, 96 and 99 As option 3 has 13 in the 3rd place it can be eliminated.
Codes for U: 01, 03, 31, 57, 77 and 95.

As option 5 has 56 in the 4th place, it can be eliminated.
Codes for C: 10, 12, 24, 42, 56, 68 and 86.
Codes for T: 00, 23, 30, 58, 76, 87 and 98.
Option 4 has all the correct codes.
Hence, option 4.

NMAT Logical Reasoning MCQ Quiz - 3 - Question 9

The question has two pairs of terms, with one term missing. Choose the term 3 that makes the relationship between the two pairs analogous.

2 : 8 :: 5 : ?

Detailed Solution for NMAT Logical Reasoning MCQ Quiz - 3 - Question 9

Since one of the answer options is “Some of the above” and one is “All of the above”, start with the assumption that there may be multiple relationships.
Now, 8 = 23 and 53 = 125.
Since 125 is present in the options, it is a possible answer. 2 x 4 = 8 and 5 x 4 = 20.
Since 20 is present in the options, it is a possible answer. 2 + 6 = 8 and 5 + 6 = 11.
Since 11 is present in the options, it is a possible answer.
Thus, all the values given in the answer options are valid to make the relationship analogous.
Hence, option 5.

NMAT Logical Reasoning MCQ Quiz - 3 - Question 10

Answer the question based on the passage given below.

In a city which has recently opened up its market to international brands, AyuJotsna Cosmetics has to now put in some serious efforts to establish its position as a market leader. The opening of the market will drastically impact its sales.


Which of the following will help AyuJotsna Cosmetics in achieving its goal?

Detailed Solution for NMAT Logical Reasoning MCQ Quiz - 3 - Question 10

Since there is no data in the passage about the economic capacity of the demographic that comprises the market, options 2 and 5 can be ruled out as they’re specific to the rich segment. ’’Segregating prices" based on the consumer demands might not guarantee achievement of market leadership for AyuJotsna Cosmetics. Eliminate option 4.
Given that the market has opened up to international brands, it is important for AyuJotsna Cosmetics to market itself as being at par with them. This can be achieved through option 1. Between options 1 and 3, option 1 is more favourable as well as feasible.
Hence, the correct answer is option 1.

NMAT Logical Reasoning MCQ Quiz - 3 - Question 11

In a round table conference of the SAARC member countries, a representative each from Afghanistan, Bangladesh, Bhutan, India, Maldives, Nepal, Pakistan and Sri Lanka sat facing each other. The Afghan sat opposite the Bangladeshi. The Bhutanese sat opposite the Indian. The Sri Lankan sat adjacent to the Bangladeshi and Indian. The Pakistani sat to the immediate right of the Afghan. The Maldivian sat at most three places to the right of the Indian.

Which of the following statements is true?

Detailed Solution for NMAT Logical Reasoning MCQ Quiz - 3 - Question 11

Since the conference was round table and the representatives sat facing each other, they were sitting around a circle facing the centre.
Number the seats 1-8 in clockwise order.
The Afghan sat opposite the Bangladeshi and the Pakistani sat to the immediate right of the Afghan.
Let the Afghan be on seat 1. So, the Bangladeshi and Pakistani were on seats 5 and 8 respectively.
Now, the Sri Lankan was adjacent to the Indian and Bangladeshi. So, there are two cases possible: Case 1: The Sri Lankan was to the right of the Bangladeshi.
So, the Indian and Sri Lankan were on seats 3 and 4 respectively.
Since the Bhutanese sat opposite the Indian, the Bhutanese was on seat 7.

Since the Maldivian sat at most three places to the right of the Indian, the Maldivian has to be to the immediate right of the Indian i.e. on seat 2. (If the Maldivian were on the only other seat remaining i.e. seat 6, he would be five places to the right of the Indian.) So, the Nepali was on seat 6.
The final arrangement in this case is as shown below: 

Case 2: The Sri Lankan was to the left of the Bangladeshi.
So, the Indian and Sri Lankan were on seats 7 and 6 respectively.
Since the Bhutanese sat opposite the Indian, the Bhutanese was on seat 3.
Since the Maldivian sat at most three places to the right of the Indian, the Maldivian could only have been on seat 4 (as seats 6 and 5 are already occupied). So, the Nepali was on seat 2. The final arrangement in this case is as shown below: 

Now, consider the statements given in the options.
Option 1: This statement is false as the Indian sat next to the Pakistani in case 2 but not in case 1.
Option 2: This statement is false as the Maldivian never sat next to the Nepali.
Option 3: This statement is false as the Nepali sat adjacent to the Bangladeshi and Bhutanese in case 1 but not in case 2.
Option 4: This statement is false as either the Pakistani or the Nepali sat opposite the Sri Lankan.
So, none of the given statements is true.
Hence, option 5.

NMAT Logical Reasoning MCQ Quiz - 3 - Question 12

Group Question

Answer the following question based on the information given below.


Three friends Ray, Bay and May eat ice cream every day at different times either in the morning or in the afternoon or at night. Each of them likes a different flavor among chocolate, vanilla and butter scotch. All of them like to eat it in a different manner using a cone, cup or biscuit. The following information is also known:

1. Ray likes chocolate ice cream.

2. Bay does not like to eat ice cream in a cup.

3. May is not fond of cones.

4. May eats cold food items during afternoon only.

5. Ray is not allowed to eat ice cream at night.  

 

 Q. Who likes Vanilla ice cream?

Detailed Solution for NMAT Logical Reasoning MCQ Quiz - 3 - Question 12

The given information can be summarized as follows: 

Either Bay or May likes Vanilla ice cream.
Hence, option 5.

NMAT Logical Reasoning MCQ Quiz - 3 - Question 13

Three friends Ray, Bay and May eat ice cream every day at different times either in the morning or in the afternoon or at night. Each of them likes a different flavor among chocolate, vanilla and butter scotch. All of them like to eat it in a different manner using a cone, cup or biscuit. The following information is also known:

1. Ray likes chocolate ice cream.

2. Bay does not like to eat ice cream in a cup.

3. May is not fond of cones.

4. May eats cold food items during afternoon only.

5. Ray is not allowed to eat ice cream at night.  

 

Q. Which of the following combinations may be true?

Detailed Solution for NMAT Logical Reasoning MCQ Quiz - 3 - Question 13

From the above table, only May - Butterscotch - Cup - afternoon may be true.
Hence, option 4.

NMAT Logical Reasoning MCQ Quiz - 3 - Question 14

The statement given below is followed by two statements. Analyze all of them and determine the correct combination of a course of action.


The recent advertising campaign has failed to make an impact on sales of a particular product.

A. The product manufacturers should launch another advertising campaign which is totally different from the first one.

B. The product manufacturers should try and determine the cause of the failure of the advertising campaign.

Detailed Solution for NMAT Logical Reasoning MCQ Quiz - 3 - Question 14

It is obvious that statement B is the most immediate course of action that the company should implement as launching a new campaign may prove to be worthless if the reasons for failure are not known.
Now, statement A would be a relevant follow-up course of action only if it is determined that the product sales had failed to pick up because the advertising campaign was ineffective. However, there can be a host of different reasons for this failure. In all these other cases, launching a new advertising campaign may again prove to be futile.
Hence, option 2.

NMAT Logical Reasoning MCQ Quiz - 3 - Question 15

Answer the following question based on the information given below.

The United Nations was the first international governmental organization to receive significant support from the United States. Its forerunner, the League of Nations, had been championed by Woodrow Wilson after World War I to prevent future conflicts. While it was supported by most European nations, it was never ratified by the United States Congress due to the inability to reach a compromise regarding the Lodge Reservations or the Hitchcock Reservations.


What conclusion can be drawn based on this passage?

Detailed Solution for NMAT Logical Reasoning MCQ Quiz - 3 - Question 15

The passage refers to the United Nations as the only body to be supported by the United States, while its forerunner, the League of Nations was not, due to its failure to reach the compromise mentioned in the passage.
There has been no discussion about the motives of the United States or the United Nations and hence, option 1 cannot be concluded.
Nothing in the passage alludes to any political connections of the League of Nations and hence, option 2 cannot be concluded.
It is obvious that the War must have had devastating consequences, which led to the formation of an organization to prevent further conflicts and hence, option 3 can be concluded.
No information has been provided about the Lodge or Hitchcock Reservations which can lead us to conclude that they were stringent and hence, option 4 can be eliminated.
No comparison between the United Nations and the League of Nations has been made and hence, option 5 can be eliminated.
Hence, the correct answer is option 3.

NMAT Logical Reasoning MCQ Quiz - 3 - Question 16

A student plays a logical game called a “perfect-misplaced” game. The rules of this game are as follows:

The student has to find out a “base word” based on certain words called “guess words” given to him. Each guess word is given with a “perfect count” and a “misplaced count”. The perfect count indicates the number of letters for which the placement of the letters in the guess word is the same as the placement of those letters in the base word. The misplaced count indicates the number of letters in the guess word that are present in the base word, but are not at the same position in both the words.

The following table shows the guess words given to the student.

 

What is the base word?

Detailed Solution for NMAT Logical Reasoning MCQ Quiz - 3 - Question 16

Consider the guess word HEAL. Since its perfect count is 3, 3 letters from this word are present in the base word and are in the same position in the base word as in HEAL.
So, the base word should have any 3 letters out of H, E, A and L; and the possible structure of the base word is HEA_ or HE_L or H AL or _EAL.
Hence, options 1 and 5 can be eliminated.
Also, because this word has no misplaced count, the fourth letter of HEAL is not present in the base word at all.
Now, consider the guess word HAND. It has a perfect count of 2 and misplaced count of 1.
This implies that 3 letters from this word are present in the base word such that 2 are in the same position as in HAND while 1 is in a position different from that in HAND.

Observe that H is in both guess words HEAL and HAND; and is in the same position in both. Thus, H is one of the letters contributing to the perfect count.
Thus, the base word starts with H.
Now observe that the letter A is common to HEAL and HAND, but is at different positions. Thus, A also has to be part of the base word and its position has to be according to the position in HEAL (because the misplaced count of HEAL is 0).
Thus, the structure of the base word is HEA_ or H_AL.

Hence, option 3 can be eliminated.
Now, consider the remaining letters of the word HAND i.e. N and D. Exactly one of these letters is in the base word. Since we have already accounted for 1 perfect count letter (H) and the only misplaced count letter (A), the letter that is present in the base word will have to be perfectly placed. N cannot be perfectly placed as the base word has A as the third letter. N cannot be misplaced at all because the only misplaced letter has already been found. So, N cannot be in the base word at all.
Thus, D is in the base word and it is the last letter of the base word.
Thus, the base word is of the form H_AD.
HEAD is the only option that satisfies this condition.
Hence, option 2.
Note: Once you eliminate option 3, you can solve the question faster by considering the remaining options i.e. HEAD and HEAT. Had HEAT been the base word, T would be perfectly placed in EAST. This would make the perfect count of EAST equal to 1. However, the perfect count of EAST is 0. So, the base word could not have been HEAT.

NMAT Logical Reasoning MCQ Quiz - 3 - Question 17

Which term from the options will continue the series?

B5, E26, H65, K122,______.

Detailed Solution for NMAT Logical Reasoning MCQ Quiz - 3 - Question 17

First consider the letters of each term, and the position of each letter in the alphabet.
B = 2, E = 5, H = 8, K = 11
Thus, the letter in each term exceeds the letter in the previous term by 3. Hence, the missing letter is 11 + 3 = 14 i.e. N Hence, options 1, 2 and 4 can be eliminated.
Now, consider the numbers. 

5 = 22 + 1 (this term has the letter B = 2)

26 = 52 + 1 (this term has the letter E = 5)

65 = 82 + 1 (this term has the letter H = 8)

122 = 112 + 1 (this term has the letter K = 11) Since the letter in the missing term is N (i.e. the 14th letter), the missing number is 142 + 1 = 196 + 1 = 197.
Thus, the missing term is N197. Hence, option 3.

NMAT Logical Reasoning MCQ Quiz - 3 - Question 18

If ‘CARDINAL’ is coded as ‘ESBDMBOJ’ and ‘SONGBIRD ’ is coded as ‘HOPTESJC’, then what is the code word for ‘GOLDFISH’?

Detailed Solution for NMAT Logical Reasoning MCQ Quiz - 3 - Question 18

The coding of the above words is done as follows: 

 

In a similar way we can do the coding of ‘GOLDFISH’ as: 

Hence, option 5.

NMAT Logical Reasoning MCQ Quiz - 3 - Question 19

125 small but identical cubes have been put together to form a large cube. How many more such small cubes will be required to cover this large cube completely?

Detailed Solution for NMAT Logical Reasoning MCQ Quiz - 3 - Question 19

125 smaller cubes can form a 5 x 5 x 5 cube. To cover it we require a 7 x 7 x 7 cube because one additional layer of cubes is required on all faces. Additional cubes required = 73 - 53 = 218. Hence, option 3.

NMAT Logical Reasoning MCQ Quiz - 3 - Question 20

A statement is followed by two assumptions. Consider the statement and the following assumptions and decide which of the assumptions is/ are implicit in the statement.

Human cloning is the creation of a genetically identical copy of a human. The term is generally used to refer to artificial human cloning, which is the reproduction of human cells and tissue. It does not refer to the natural conception and delivery of identical twins. The possibility of human cloning has raised controversies. These ethical concerns have prompted several nations to pass laws regarding human cloning and its legality.


Assumptions:

A. Human cloning is an upcoming field and has potential.

B. We live in an age dominated by the thought that every human is unique.

Detailed Solution for NMAT Logical Reasoning MCQ Quiz - 3 - Question 20

The passage focuses mainly on the moral and ethical aspects of human cloning, without getting into its technological merits and hence, we cannot make assumptions on its potential. Assumption 1 is not implicit in the passage and can be eliminated. Eliminate options 1, 3 and 5.
The passage states that “Human cloning is the creation of a genetically identical copy of a human.”. The creation of a copy of a human destroys the exclusivity attributed to each human and hence, assumption B is not implicit. Moreover, the passage does not delve into the reason why human cloning has raised controversies. Eliminate option 2.
Therefore, none of these assumptions are implicit.
Hence, the correct answer is option 4.

NMAT Logical Reasoning MCQ Quiz - 3 - Question 21

Answer the question based on the passage given below.

Many food stores in the city have begun to stock ready-to-cook meals. Therefore, getting food ready in a short time has become important to people.

 

Q. A statement that can strengthen the author’s conclusion is

Detailed Solution for NMAT Logical Reasoning MCQ Quiz - 3 - Question 21

Option 1 weakens the argument; it presents an inadequacy in the evidence (Many food stores in the city have begun to stock ready-to-cook meals) that stocking may not be equivalent to higher sales.
Option 2 does not necessarily strengthen the argument - if the most urbanized places in the city are very few, then not too many stores with ready-to-cook meals may be present in the city.
Option 3 strengthens the argument; higher sales do indicate a higher preference for ready-to-cook meals.
Option 4 does not strengthen the argument; a boom in sales of vegetables and fruits may or may not be related to higher sales or preference of ready-to- cook meals.
Option 5 neither strengthens nor weakens the argument- even if people are unable to buy these meals due to the expense involved, they may still wish to. The cost factor may be the only thing stopping them.
Hence, the correct answer is option 3.

NMAT Logical Reasoning MCQ Quiz - 3 - Question 22

1 3 7 13 21 31 43

Which of the following is the next number in the series given above?

Detailed Solution for NMAT Logical Reasoning MCQ Quiz - 3 - Question 22

1 3 7 13 21 31 43
The above series follows the logic: The nth term, Tn = n2 - (n - 1) Since we want 8th term in the series we put n = 8 Next number in the series = 64 - (8 - 1) = 57

Hence, option 2.

NMAT Logical Reasoning MCQ Quiz - 3 - Question 23

Which number will replace x?

Detailed Solution for NMAT Logical Reasoning MCQ Quiz - 3 - Question 23

Starting from the box at the top left and going clockwise, the sum of numbers in each box is 27, 24 and 21.
Since these are consecutively decreasing multiples of 3, the sum of numbers in the box at the bottom left should be 18  x + 6 + 7 + 4= 18. x = 1
Hence, option 5.

NMAT Logical Reasoning MCQ Quiz - 3 - Question 24

In a certain language, if COLORS is coded as XLOLIH, how is PICTURE coded in that language?

Detailed Solution for NMAT Logical Reasoning MCQ Quiz - 3 - Question 24

Consider the alphabetical position of each letter of COLORS and XLOLIH.

C O  L  O   R    S
3 15 12 15 18 19
 X  L   O   L    I    H 
 24 12 15 12 9   8

Thus, in each case, the sum of alphabetical position of a letter and its respective code is 27.
Now, consider the alphabetical position of each letter of PICTURE.
P= 16,1 = 9, C = 3,T = 20, U = 21, R= 18 and E = 5 Hence, the position of the coded letters is 11, 18, 24, 7, 6, 9, 22 i.e. PICTURE is coded as KRXGFIV.
Hence, option 5.

NMAT Logical Reasoning MCQ Quiz - 3 - Question 25

Given below are two statements. Analyze them and mark the option that correctly states their relationship.

A. The press release indicates that Saurav Kookreja has received anticipatory bail.

B. The stock price of Mr. Saurav Kookreja’s company has stabilized after plummeting for over two weeks. 

Detailed Solution for NMAT Logical Reasoning MCQ Quiz - 3 - Question 25

The two statements here can have a cause-effect relationship based on certain assumptions. If these assumptions are valid in most cases, then the relationship holds true.
For statement B to be an effect of the cause stated in statement A, it should be given that Mr. Kookreja’s bail was the primary factor in the stabilization of his company’s stock price. This is fairly simple to assume since it is also given that the price had been falling for the last two weeks (till the news of his bail was not public). Other factors, if any, are clearly overshadowed by this one major factor. Hence, it is safe to say that A is the cause of B.
Hence, the correct answer is option 1.

NMAT Logical Reasoning MCQ Quiz - 3 - Question 26

Answer the following question based on the information given below.


The laws of Abode were very strict in regard to associations, which, formed on the pretense of amusement, charity, or athletic sports, were apt to degenerate into political sects. Exception was made in favor of the collegia funeraticia, which were societies formed to provide a decent funeral and place of burial for their members. An inscription discovered at Civita Lavinia quotes the very words of a decree of the Senate on this subject: "It is permitted to those who desire to make a monthly contribution for funeral expenses to form an association." "These clubs or colleges collected their subscriptions in a treasure-chest, and out of it provided for the obsequies of deceased members. Funeral ceremonies did not cease when the body or the ashes were laid in the tomb. It was the custom to celebrate on the occasion a feast, and to repeat that feast year by year on the birthday of the dead, and on other stated days. For the holding of these feasts, as well as for other meetings, special buildings were erected, named scholae and when the societies received gifts from rich members or patrons, the benefaction frequently took the shape of a new lodge-room, or of a ground for a new cemetery, with a building for meetings." The Holiacs took advantage of the  freedom accorded to funeral colleges, and associated themselves for the same purpose, following as closely as possible their rules concerning contributions, the erection of lodges, the meetings, and the love feasts; and it was largely through the adoption of these well-understood and respected customs that they were enabled to hold their meetings and keep together as a corporate body through the stormy times of the second and third centuries.

 

 

Q. Based on the above passage, which of the following is/ are most likely to be true?
I. For the populace of Athens, getting involved in amusement, charity or athletic activities was dangerous.
II. Funeral colleges were responsible for funeral ceremonies - the family was not.
III. The emulation of funeral colleges by Holiacs allowed them to survive the second and third centuries.

Detailed Solution for NMAT Logical Reasoning MCQ Quiz - 3 - Question 26

There is no data on “Athens”. Eliminate all options except 2.
Statement II takes away the entire responsibility of the ceremonies from “the family” - for which there is no justification in the passage.
By emulating the practices of the funeral colleges, the “Holiacs” managed to hold meetings and keep together as a community. Therefore, statement III holds.
Hence, the correct answer is option 2.

NMAT Logical Reasoning MCQ Quiz - 3 - Question 27

Which of the following numbers will replace the question mark? 

Detailed Solution for NMAT Logical Reasoning MCQ Quiz - 3 - Question 27

The numbers in each row and column add up to 18 Hence, if the missing number is x, 3 + x + 9 = 18 i.e. x = 6 Hence, option 4.

NMAT Logical Reasoning MCQ Quiz - 3 - Question 28

In the question given below, the main statement is followed by three assumptions. Choose the option that states the ones that are implicit. Groups that comprise selfish organisms have been observed to go extinct.


I. Within a group, nonselfish organisms have a selective individual advantage relative to their selfish colleagues.

II. The survival fitness of a group as a whole is decreased by the presence of selfish organisms.

III. Groups comprising nonselfish organisms are likely to have a survival advantage over groups comprising selfish organisms.

Detailed Solution for NMAT Logical Reasoning MCQ Quiz - 3 - Question 28

According to the main statement groups having a larger number of selfish organisms do not survive. This implies that such groups have lesser survival fitness. Thus, statement II is an implicit assumption. Eliminate options 1 and 5. The statement talks only about the fate of the group as a whole; nothing has been said about the fate of the individuals within the group as being relative to one another. Therefore, the assumption made in statement I is not implied by the main statement. Eliminate option 4.
If groups having a larger number of selfish individuals do not survive, then this implies that groups that have no selfish individuals or a smaller number of them do survive. Therefore, statement III also is an implicit assumption. Eliminate option 2.
Hence, the correct answer is option 3.

NMAT Logical Reasoning MCQ Quiz - 3 - Question 29

In a certain code, if GRIP is coded as I7T18K9R16, how is FADE coded in the same code?

Detailed Solution for NMAT Logical Reasoning MCQ Quiz - 3 - Question 29

There are two patterns applied to each original letter of the word.
G + 2 = I and G = 7;R + 2 = T and R =18; I + 2 = K and I = 9; P + 2 = R andP = 16 Thus, each letter is replaced by the letter that is two places ahead in the alphabet as well as by its own position in the alphabet.
Apply this logic to each letter of FADE.
F + 2 = H and F = 6;A + 2 = C and A =1; D + 2 = F and D = 4;E + 2 = G and E = 5 Thus, FADE is coded as H6C1F4G5.
Hence, option 2.

NMAT Logical Reasoning MCQ Quiz - 3 - Question 30

Group Question

Answer the following question based on the information given below.


Some data is fed is into a computing machine. It processes the input and gives an output in the following manner:

Input: AE017 392MC P2G12 18S2B 20UL3

Step 1: AE017 CM239 GP122 BS128 LU023

Step 2: AE025 CM253 GP127 BS139 LU028

Step 3: AE025 CM235 GP127 BS139 LU028

Step 4: AE025 BS139 CM235 GP127 LU028

 

Step 4 is the final output of the input processed.

 

Q. What would be Step 4 for the following input?

Input: M92G1 DE625 AK117 OJ120 HC224

Detailed Solution for NMAT Logical Reasoning MCQ Quiz - 3 - Question 30

First find the pattern applied to the original input.
Step 1: In each term, the letters are arranged followed by the numbers. Also, the letters are in alphabetical order while the numbers are in ascending order.
Step 2: The sum of digits of the numbers is added to that respective number in each term.
Step 3: For each term, the digits are arranged in ascending order.
Step 4: Consider the first letter of each term, the terms are arranged in ascending order.
Applying this logic to the given input:

Input: M92G1 DE625 AK117 OJ120 HC224

Step 1: GM129 DE256 AK117 JO012 CH224

Step 2: GM141 DE269 AK126 JO015 CH232

Step 3: GM114 DE269 AK126 JO015 CH223

Step 4: AK126 CH223 DE269 GM114 JO015 Hence, option 4.  

View more questions
1 docs|29 tests
Information about NMAT Logical Reasoning MCQ Quiz - 3 Page
In this test you can find the Exam questions for NMAT Logical Reasoning MCQ Quiz - 3 solved & explained in the simplest way possible. Besides giving Questions and answers for NMAT Logical Reasoning MCQ Quiz - 3, EduRev gives you an ample number of Online tests for practice

Top Courses for CAT

Download as PDF

Top Courses for CAT